Vector calculus - How to use the gradient?

Click For Summary
The discussion centers on using the gradient to solve problems related to the function T = ln(x^2 + y^2) at the point (1,2). The user has successfully calculated the gradient, which indicates the direction of the most rapid increase in T as (2/5, 4/5). For part (b), the approach involves determining the distance in that direction that results in a specific increase in T, using the relationship ΔT = GradT · r. Part (c) requires a similar method but focuses on the direction of (1, 1) for a different increase in T, while part (d) involves finding stationary points where the gradient is perpendicular to the direction vector. The user seeks guidance on how to approach these problems without needing the full solutions.
alexvenk
Messages
11
Reaction score
0
Member warned about posting without the template
I have done part A (i think) really not sure where to begin with the rest of the parts, would appreciate a tip in the right direction, its revision for my first year physics exams in a few weeks.

Consider the funtion T in the plane (x,y), given by T=ln(x^2 + y^2)

at point 1,2

a) in which direction is most rapid increase in T

I did Grad(T) to get a vector which i think is in the direction of most rapid increase (2/5,4/5)

b) what distance in this direction gives an inrease of .2 in T

c) what distance in direction i + j gives and increase of .12 in T

d) in what directions will T be stationary.

I don't want solutions, just how to go about solving the problems
 
Physics news on Phys.org
I have done part A (i think) really not sure where to begin with the rest of the parts, would appreciate a tip in the right direction, its revision for my first year physics exams in a few weeks.

Consider the funtion T in the plane (x,y), given by T=ln(x^2 + y^2)

at point 1,2

a) in which direction is most rapid increase in T

I did Grad(T) to get a vector which i think is in the direction of most rapid increase (2/5,4/5)

b) what distance in this direction gives an inrease of .2 in T

c) what distance in direction i + j gives and increase of .12 in T

d) in what directions will T be stationary.

I don't want solutions, just how to go about solving the problems
 
alexvenk said:
I have done part A (i think) really not sure where to begin with the rest of the parts, would appreciate a tip in the right direction, its revision for my first year physics exams in a few weeks.

Consider the funtion T in the plane (x,y), given by T=ln(x^2 + y^2)

at point 1,2

a) in which direction is most rapid increase in T

I did Grad(T) to get a vector which i think is in the direction of most rapid increase (2/5,4/5)

b) what distance in this direction gives an inrease of .2 in T

c) what distance in direction i + j gives and increase of .12 in T

d) in what directions will T be stationary.

I don't want solutions, just how to go about solving the problems

For (b): if you go along direction (2/5,4/5) from the point (1,2) you are looking at points of the form ##x = x(t) = 1 + (2/5)t, \:y = y(t) = 2 + (4/5)t##, where ##t > 0## is a scalar.
 
(DelT = delta T (change in T))

Turns out the best way to do it for those who are interested is you use DelT = GradT . r, to get the largest change in t (highest delT) r and GradT must be in the same direction. To work out how far in a certain direcction it changes by a certain amount, set delT to whatever you want the change to be (.2 for b) then set r to be a vecctor with magnitude a and direction the same as the direction it was in a, then simply solve for a. Do the same for part c, and finally for part d, set delT to 0 so GradT must be perpendicular to r, which is pretty easy to do by inspection.

Thanks for the reply.
 
For simplicity, direction of (2/5,4/5) is the same of (1,2).
 
Question: A clock's minute hand has length 4 and its hour hand has length 3. What is the distance between the tips at the moment when it is increasing most rapidly?(Putnam Exam Question) Answer: Making assumption that both the hands moves at constant angular velocities, the answer is ## \sqrt{7} .## But don't you think this assumption is somewhat doubtful and wrong?

Similar threads

Replies
7
Views
2K
  • · Replies 9 ·
Replies
9
Views
2K
  • · Replies 8 ·
Replies
8
Views
2K
  • · Replies 8 ·
Replies
8
Views
2K
  • · Replies 8 ·
Replies
8
Views
2K
Replies
9
Views
2K
  • · Replies 8 ·
Replies
8
Views
2K
  • · Replies 2 ·
Replies
2
Views
2K
  • · Replies 15 ·
Replies
15
Views
2K
Replies
12
Views
9K